Trigonometry

Geometry Level 3

tan 9 tan 2 7 tan 6 3 + tan 8 1 = ? \large \tan9^\circ-\tan27^\circ-\tan63^\circ+\tan81^\circ = \, ?


The answer is 4.

This section requires Javascript.
You are seeing this because something didn't load right. We suggest you, (a) try refreshing the page, (b) enabling javascript if it is disabled on your browser and, finally, (c) loading the non-javascript version of this page . We're sorry about the hassle.

2 solutions

Rishabh Jain
Jul 7, 2016

How many times will this question be repeated on brilliant?

( tan 8 1 + tan 9 ) ( tan 6 3 + tan 2 7 ) (\tan 81^\circ+\tan9^\circ)-(\tan 63^\circ+\tan27^\circ)

= ( sin 9 0 1 cos 9 cos 8 1 sin 9 sin 1 8 2 ) ( sin 9 0 1 cos 2 7 cos 6 3 sin 2 7 sin 5 4 2 ) =\left(\dfrac{\overbrace{\sin 90^{\circ}}^{\color{#D61F06}{1}}}{\underbrace{\cos 9^{\circ}\underbrace{\cos 81^{\circ}}_{\sin9^{\circ}}}_{\color{#D61F06}{\dfrac{\sin 18^{\circ}}{2}}}}\right)-\left( \dfrac{\overbrace{\sin 90^{\circ}}^{\color{#D61F06}{1}}}{\underbrace{\cos 27^{\circ}\underbrace{\cos 63^{\circ}}_{\sin 27^{\circ}}}_{\color{#D61F06}{\dfrac{\sin 54^{\circ}}2}}}\right)

= 2 ( 1 5 1 4 1 5 + 1 4 ) \large=2\left(\dfrac{1}{\frac{\sqrt 5-1}{4}}-\dfrac{1}{\frac{\sqrt 5+1}{4}}\right)

= 2 × 2 = 4 \huge =2\times 2=\boxed{\color{#3D99F6}{4}}


In second line I used : tan A + tan B = sin ( A + B ) cos A cos B , sin A cos A = sin 2 A 2 \small{\color{teal}{\tan A+\tan B=\dfrac{\sin (A+B)}{\cos A\cos B}~~,\sin A\cos A=\dfrac{\sin 2A}2}}

Also, sin 1 8 = 5 1 4 , sin 5 4 = 5 + 1 4 \small{\sin 18^{\circ}=\dfrac{\sqrt 5-1}{4},\sin 54^{\circ}=\dfrac{\sqrt 5+1}{4}}

how do you know the value of sin 18 and sin 54 ??

sanyam goel - 4 years, 11 months ago

Log in to reply

Let x=18, 5x=90, 3x=90-2x .. Take sin on both sides and use sin 3x =3sin x-4sin^3 x and cos 2x=1-2sin^2 x to solve for sin x... Having found sin 18 it is trivial to found cos 36(=sin 54) by using 1-2sin^2 18....

Rishabh Jain - 4 years, 11 months ago

X = tan 9 tan 2 7 tan 6 3 + tan 8 1 = tan 9 + tan 8 1 tan 2 7 tan 6 3 = tan ( 4 5 3 6 ) + tan ( 4 5 + 3 6 ) tan ( 4 5 1 8 ) tan ( 4 5 + 1 8 ) = 1 tan 3 6 1 + tan 3 6 + 1 + tan 3 6 1 tan 3 6 1 tan 1 8 1 + tan 1 8 1 + tan 1 8 1 tan 1 8 = ( 1 tan 3 6 ) 2 + ( 1 + tan 3 6 ) 2 1 tan 2 3 6 ( 1 tan 1 8 ) 2 + ( 1 + tan 1 8 ) 2 1 tan 2 1 8 = 2 ( 1 + tan 2 3 6 ) 2 ( 1 tan 2 3 6 ) 2 ( 1 + tan 2 1 8 ) 2 ( 1 tan 2 1 8 ) Note that: cos 2 θ = 1 tan 2 θ 1 + tan 2 θ = 2 cos 7 2 2 cos 3 6 = 2 ( cos 3 6 cos 7 2 ) cos 7 2 cos 3 6 Note that: cos ( 18 0 θ ) = cos θ = 2 ( cos 3 6 + cos 10 8 ) cos 7 2 cos 3 6 Note that: k = 0 n 1 cos ( 2 k + 1 ) π ( 2 n + 1 ) = 1 2 = 1 cos 7 2 cos 3 6 = sin 3 6 cos 7 2 cos 3 6 sin 3 6 = 2 sin 3 6 cos 7 2 sin 7 2 = 4 sin 3 6 sin 14 4 Note that: sin ( 18 0 θ ) = sin θ = 4 sin 3 6 sin 3 6 = 4 \begin{aligned} \mathscr X & = \tan 9^\circ - \tan 27^\circ - \tan 63^\circ + \tan 81^\circ \\ & = \tan 9^\circ + \tan 81^\circ - \tan 27^\circ - \tan 63^\circ \\ & = \tan (45^\circ-36^\circ) + \tan (45^\circ+36^\circ) - \tan (45^\circ-18^\circ) - \tan (45^\circ+18^\circ) \\ & = \frac {1 - \tan 36^\circ}{1 + \tan 36^\circ} + \frac {1 + \tan 36^\circ}{1 - \tan 36^\circ} - \frac {1 - \tan 18^\circ}{1 + \tan 18^\circ} - \frac {1 + \tan 18^\circ}{1 - \tan 18^\circ} \\ & = \frac {(1 - \tan 36^\circ)^2 + (1 + \tan 36^\circ)^2}{1 - \tan^2 36^\circ} - \frac {(1 - \tan 18^\circ)^2 + (1 + \tan 18^\circ)^2}{1 - \tan^2 18^\circ} \\ & = \frac {2(1 + \tan^2 36^\circ)}{2(1 - \tan^2 36^\circ)} - \frac {2(1 + \tan^2 18^\circ)}{2(1 - \tan^2 18^\circ)} \quad \quad \small \color{#3D99F6}{\text{Note that: }\cos 2 \theta = \frac {1-\tan^2 \theta}{1+ \tan^2 \theta}} \\ & = \frac 2{\cos 72^\circ} - \frac 2{\cos 36^\circ} \\ & = \frac {2(\cos 36^\circ \color{#3D99F6}{- \cos 72^\circ})}{\cos 72^\circ \cos 36^\circ} \quad \quad \small \color{#3D99F6}{\text{Note that: }\cos (180^\circ - \theta) = - \cos \theta} \\ & = \frac {2(\cos 36^\circ \color{#3D99F6}{+ \cos 108^\circ})}{\cos 72^\circ \cos 36^\circ} \quad \quad \small \color{#3D99F6}{\text{Note that: }\sum_{k=0}^{n-1} \cos \frac {(2k+1)\pi}{(2n+1)} = \frac 12} \\ & = \frac 1{\cos 72^\circ \cos 36^\circ} \\ & = \frac {\sin 36^\circ}{\cos 72^\circ \cos 36^\circ \sin 36^\circ} \\ & = \frac {2 \sin 36^\circ}{\cos 72^\circ \sin 72^\circ} \\ & = \frac {4 \color{#3D99F6}{\sin 36^\circ}}{\color{#3D99F6}{\sin 144^\circ}} \quad \quad \small \color{#3D99F6}{\text{Note that: }\sin (180^\circ - \theta) = - \sin \theta} \\ & = \frac {4 \color{#3D99F6}{\sin 36^\circ}}{\color{#3D99F6}{\sin 36^\circ}} \\ & = \boxed{4} \end{aligned}

0 pending reports

×

Problem Loading...

Note Loading...

Set Loading...